Finding the Total Angular Momentum

In summary, you first find the (orbital+spin)angular momentum for the particles using the jj coupling scheme. You then find the total angular momentum of the system by summing the square of the coefficients of the individual angular momentum vectors.
  • #1
devd
47
1
Say, I have two spin-1/2 particles in the states characterized by ##(n=2, l=1, m_l=1, m_s=1/2)##and##(n=2, l=1, m_l=1, m_s=-1/2)##. Now, using something like the jj coupling scheme, I first find out the (orbital+spin)angular momentum for the individual particles:(i) ##| 11\rangle |\frac{1}{2}-\frac{1}{2}\rangle =\sqrt{1/3}| \frac{3}{2}\frac{1}{2}\rangle+\sqrt{2/3}|\frac{1}{2}\frac{1}{2}\rangle##

(ii)##|11\rangle|\frac{1}{2}\frac{1}{2}\rangle=|\frac{3}{2}\frac{3}{2}\rangle##

How do i proceed to find the total angular momentum of the system?

I've tried to add like this:
##\Big(| 11\rangle |\frac{1}{2}-\frac{1}{2}\rangle\Big)\Big(|11\rangle|\frac{1}{2}\frac{1}{2}\rangle\Big)= \sqrt{1/6}|3,2\rangle+\Big(\sqrt{2/3}-\sqrt{1/6}\Big)|2,2\rangle##

But, the sum of the square of the coefficients don't add up to 1! So, where did i go wrong?
 
Physics news on Phys.org
  • #2
The reason it doesn't work is because the notation ##|\psi_1\rangle |\psi_2\rangle## does not really mean ##|\psi_1\rangle ## being algebraically multiplied with ##|\psi_2\rangle##. Notation like ##|\psi_1\rangle |\psi_2\rangle## merely means that, that state is a member of a composite vector space constituted from individual vector spaces in which ##|\psi_1\rangle## and ##|\psi_2\rangle## are respectively members.
 
  • #3
blue_leaf77 said:
The reason it doesn't work is because the notation ##|\psi_1\rangle |\psi_2\rangle## does not really mean ##|\psi_1\rangle ## being algebraically multiplied with ##|\psi_2\rangle##. Notation like ##|\psi_1\rangle |\psi_2\rangle## merely means that, that state is a member of a composite vector space constituted from individual vector spaces in which ##|\psi_1\rangle## and ##|\psi_2\rangle## are respectively members.
We're basically taking the tensor product, right? The tensor product is associative. So, in the last step I've basically tried to take the tensor product of product spaces. How does one do that correctly?
 
  • #4
You are basically summing 4 angular momenta ##\mathbf{L}_1##, ##\mathbf{S}_1##, ##\mathbf{L}_2##, and ##\mathbf{S}_2##. I can't think of any better way to do it other than using matrix operations. Since you want to express the state in the form ##|j,m \rangle##, you need to solve the eigenstates of both the total (squared) angular momentum and its z component:
$$
J^2 = ( \mathbf{L}_1\otimes\mathbf{1}_{S1}\otimes\mathbf{1}_{L2}\otimes\mathbf{1}_{S2} + \mathbf{1}_{L1}\otimes\mathbf{S}_1\otimes\mathbf{1}_{L2}\otimes\mathbf{1}_{S2} + \mathbf{1}_{L1}\otimes\mathbf{1}_{S1}\otimes\mathbf{L}_2\otimes\mathbf{1}_{S2} + \mathbf{1}_{L1}\otimes\mathbf{1}_{S1}\otimes\mathbf{1}_{L2}\otimes\mathbf{S}_2 )^2
$$
and
$$
J_z = L_{1z}\otimes\mathbf{1}_{S1}\otimes\mathbf{1}_{L2}\otimes\mathbf{1}_{S2} + \mathbf{1}_{L1}\otimes S_{1z}\otimes\mathbf{1}_{L2}\otimes\mathbf{1}_{S2} + \mathbf{1}_{L1}\otimes\mathbf{1}_{S1}\otimes L_{2z}\otimes\mathbf{1}_{S2} + \mathbf{1}_{L1}\otimes\mathbf{1}_{S1}\otimes\mathbf{1}_{L2}\otimes S_{2z}
$$
where the notation like ##\mathbf{1}_{V}## means an identity matrix whose dimension is the same as that of a space ##V##. Those expressions may look nasty, but if you have matrix based calculator, such as matlab, it can help your work considerably. Note that, both matrices ##J^2## and ##J_z## should be diagonalized by the same unitary matrix, indicating that they shared the same set of eigenstates. This means, you can, for example try to find the eigenstates (and hence, the eigenvalues) of ##J_z##, and then use the unitary matrix formed by these newly found eigenstates to diagonalize ##J^2##. Having done all these obviously tedious steps, you will have ##Dim[L_1]\times Dim[S_1]\times Dim[L_2]\times Dim[S_2]## equations, each representing an eigenstate ##|j,m\rangle## of ##J_z## (and ##J^2##) as a linear combination of the old eigenstates ##|l_1,m_{l1}\rangle |s_1,m_{s1}\rangle |l_2,m_{l2}\rangle |s_2,m_{s2}\rangle##. Then solve this system of linear equations for your original state in question. You should see that your original state is a linear combination of several ##|j,m\rangle##'s.
 
  • Like
Likes devd

Related to Finding the Total Angular Momentum

1. What is total angular momentum?

Total angular momentum is a physical quantity that describes the rotational motion of an object. It is a vector quantity that takes into account the mass, shape, and velocity of an object as it rotates around an axis.

2. How is total angular momentum calculated?

Total angular momentum is calculated by multiplying the moment of inertia (a measure of an object's resistance to rotational motion) by the angular velocity (the rate at which the object is rotating) and the direction of rotation. This can be represented by the equation L = Iω, where L is total angular momentum, I is moment of inertia, and ω is angular velocity.

3. What are the units of total angular momentum?

Total angular momentum is typically measured in units of kilogram meters squared per second (kg·m²/s), but can also be expressed in other units such as joule-seconds (J·s) or newton-meters per second (N·m/s).

4. What is the conservation of total angular momentum?

The conservation of total angular momentum is a fundamental principle in physics that states that the total angular momentum of a closed system remains constant over time. This means that if an object's angular momentum changes, then the angular momentum of another object in the system will change in the opposite direction to maintain the total angular momentum.

5. How is total angular momentum related to rotational kinetic energy?

Total angular momentum is directly related to rotational kinetic energy, as both quantities are measures of an object's rotational motion. The higher an object's total angular momentum, the more rotational kinetic energy it possesses. This relationship is described by the equation E = ½Iω², where E is rotational kinetic energy, I is moment of inertia, and ω is angular velocity.

Similar threads

  • Quantum Physics
Replies
4
Views
636
Replies
2
Views
832
Replies
2
Views
732
Replies
10
Views
1K
  • Quantum Physics
Replies
15
Views
2K
  • Quantum Physics
Replies
1
Views
754
  • Quantum Physics
Replies
8
Views
880
Replies
16
Views
414
Replies
5
Views
1K
Replies
3
Views
486
Back
Top